K
Khách

Hãy nhập câu hỏi của bạn vào đây, nếu là tài khoản VIP, bạn sẽ được ưu tiên trả lời.

4 tháng 4 2018

1) cho a = cănbâc3(căn5+2) -cănbâcba(căn5-2)
tính giá trị biếu thức
a^5 +4a^3 - 4a^2 +3a
2) tìm t =5/x- x/4 biết x thỏa mãn
thỏa mãn pt
x^2 /4 +100/x^2 =35+120/x -6x
3) tìm các số nguyên dương
3x^2 -18y^2 +2z^2 +3y^2z^2 -18x =27
4/ giải phương trình
x^2 =căn (x^3 -x )+ căn(x^2 -x)
5) tìm a hai phưng trình ẩn x thỏa mãn
x^2 +x +a=0 và x^2 +ax +1=0
a)
có nghiệm chung
b) hpt tương đương
6/tim hai số m; n thuộc N sao cho x thuộc N
m^2 +n^2 +mn =3x

25 tháng 7 2019

từ câu a) ta có: \(\orbr{\begin{cases}x=y+1\\x=y-1\end{cases}}\) và \(\hept{\begin{cases}x-y=t-z\\y=t\end{cases}}\) (3) 

+) Với \(x=y+1\) thì (3) \(\Leftrightarrow\)\(\hept{\begin{cases}y+1-y=y-z\\y=t\end{cases}}\Leftrightarrow\hept{\begin{cases}y=z+1\\y=t\end{cases}}\)

\(\Rightarrow\)\(x=y+1=z+2\) ( x,y,z là 3 số nguyên liên tiếp ) 

+) Với \(x=y-1\) thì (3) \(\Leftrightarrow\)\(\hept{\begin{cases}y-1-y=y-z\\y=t\end{cases}}\Leftrightarrow\hept{\begin{cases}y=z-1\\y=t\end{cases}}\)

\(\Rightarrow\)\(x=y-1=z-2\) ( x,y,z là 3 số nguyên liên tiếp ) 

25 tháng 7 2019

\(x+z=y+t\)\(\Leftrightarrow\)\(x^2+z^2+2xz=y^2+t^2+2yt\) (1) 

Mà \(xz+1=yt\)\(\Leftrightarrow\)\(2xz+2=2yt\)

(1) \(\Leftrightarrow\)\(x^2+z^2+2yt=y^2+t^2+2xz+4\)

\(\Leftrightarrow\)\(\left(x-z\right)^2-\left(y-t\right)^2=4\)

\(\Leftrightarrow\)\(\left(x-z-y+t\right)\left(x-z+y-t\right)=4\) (2) 

Lại có: \(x+z=y+t\)\(\Rightarrow\)\(\hept{\begin{cases}x-y=t-z\\x-t=y-z\end{cases}}\)

(2) \(\Leftrightarrow\)\(\left(x-y\right)\left(x-t\right)=1\)

TH1: \(\hept{\begin{cases}x-y=1\\x-t=1\end{cases}}\Leftrightarrow\hept{\begin{cases}x=y+1\\x=t+1\end{cases}}\Leftrightarrow y=t\)

TH2: \(\hept{\begin{cases}x-y=-1\\x-t=-1\end{cases}}\Leftrightarrow\hept{\begin{cases}x=y-1\\x=t-1\end{cases}}\Leftrightarrow y=t\)

1 tháng 5 2020

Ta có:

\(\frac{1}{a}+\frac{1}{b}=\frac{1}{c}\Leftrightarrow\left(a+b\right)c=ab\Leftrightarrow ab-bc-ab=0\)

Hay \(ab-bc-ab+c^2=c^2\Leftrightarrow\left(b-c\right)\left(a-c\right)=c^2\)

Nếu \(\left(b-c;a-c\right)=d\ne1\Rightarrow c^2=d^2\left(loai\right)\)

Vậy \(\left(b-c;a-c\right)=1\Rightarrow c-b;c-a\) là 2 số chính phương

Đặt \(b-c=n^2;a-c=m^2\)

\(\Rightarrow a+b=b-c+a-c+2c=m^2+n^2+2mn=\left(m+n\right)^2\) là số chính phương

27 tháng 3 2020

Bài 1 : 

Phương trình <=> 2x . x2 = ( 3y + 1 ) + 15

Vì \(\hept{\begin{cases}3y+1\equiv1\left(mod3\right)\\15\equiv0\left(mod3\right)\end{cases}\Rightarrow\left(3y+1\right)^2+15\equiv1\left(mod3\right)}\)

\(\Rightarrow2^x.x^2\equiv1\left(mod3\right)\Rightarrow x^2\equiv1\left(mod3\right)\)

( Vì số  chính phương chia 3 dư 0 hoặc 1 ) 

\(\Rightarrow2^x\equiv1\left(mod3\right)\Rightarrow x\equiv2k\left(k\inℕ\right)\)

Vậy \(2^{2k}.\left(2k\right)^2-\left(3y+1\right)^2=15\Leftrightarrow\left(2^k.2.k-3y-1\right).\left(2^k.2k+3y+1\right)=15\)

Vì y ,k \(\inℕ\)nên 2k . 2k + 3y + 1 > 2k .2k - 3y-1>0

Vậy ta có các trường hợp: 

\(+\hept{\begin{cases}2k.2k-3y-1=1\\2k.2k+3y+1=15\end{cases}\Leftrightarrow\hept{\begin{cases}2k.2k=8\\3y+1=7\end{cases}\Rightarrow}k\notinℕ\left(L\right)}\)

\(+,\hept{\begin{cases}2k.2k-3y-1=3\\2k.2k+3y+1=5\end{cases}\Leftrightarrow\hept{\begin{cases}2k.2k=4\\3y+1=1\end{cases}\Rightarrow}\hept{\begin{cases}k=1\\y=0\end{cases}\left(TM\right)}}\)

Vậy ( x ; y ) =( 2 ; 0 ) 

27 tháng 3 2020

Bài 3: 

Giả sử \(5^p-2^p=a^m\)    \(\left(a;m\inℕ,a,m\ge2\right)\)

Với \(p=2\Rightarrow a^m=21\left(l\right)\)

Với \(p=3\Rightarrow a^m=117\left(l\right)\)

Với \(p>3\)nên p lẻ, ta có

\(5^p-2^p=3\left(5^{p-1}+2.5^{p-2}+...+2^{p-1}\right)\Rightarrow5^p-2^p=3^k\left(1\right)\)    \(\left(k\inℕ,k\ge2\right)\)

Mà \(5\equiv2\left(mod3\right)\Rightarrow5^x.2^{p-1-x}\equiv2^{p-1}\left(mod3\right),x=\overline{1,p-1}\)

\(\Rightarrow5^{p-1}+2.5^{p-2}+...+2^{p-1}\equiv p.2^{p-1}\left(mod3\right)\)

Vì p và \(2^{p-1}\)không chia hết cho 3 nên \(5^{p-1}+2.5^{p-2}+...+2^{p-1}⋮̸3\)

Do đó: \(5^p-2^p\ne3^k\), mâu thuẫn với (1). Suy ra giả sử là điều vô lý

\(\rightarrowĐPCM\)

8 tháng 1 2021

Đặt \(\dfrac{x}{z}=a;\dfrac{y}{z}=b\).

Theo gt ta có \(a+b\le1\).

BĐT cần chứng minh tương đương:

\(a^2+b^2+\frac{a^2}{b^2}+\frac{b^2}{a^2}+\frac{1}{a^2}+\frac{1}{b^2}\ge \frac{21}{2}\).

Theo bđt AM - GM: \(\dfrac{a^2}{b^2}+\dfrac{b^2}{a^2}\ge2;a^2+\dfrac{1}{16}a^2\ge\dfrac{1}{2};b^2+\dfrac{1}{16}b^2\ge\dfrac{1}{2};\dfrac{15}{16}\left(\dfrac{1}{a^2}+\dfrac{1}{b^2}\right)\ge\dfrac{15}{32}\left(\dfrac{1}{a}+\dfrac{1}{b}\right)^2\ge\dfrac{15}{32}.\left(\dfrac{4}{a+b}\right)^2\ge\dfrac{15}{2}\).

Cộng vế với vế của các bđt trên lại ta có đpcm.

 

5 tháng 2 2019

xy+yz+xz=3xyz

<=> xy+yz+xz/xyz = 3

<=> 1/x + 1/y + 1/z = 3

Do vai trò x ; y ; z như nhau , ko mất tính tổng quát , giả sử 

\(x\ge y\ge z\) . Khi đó , ta có : 

\(\frac{1}{x}+\frac{1}{y}+\frac{1}{z}\le3.\frac{1}{x}\)

\(\Rightarrow3\le3.\frac{1}{x}\)

\(\Rightarrow1\le\frac{1}{x}\)

\(\Rightarrow x\le1\)

Mà x nguyên dương nên x = 1

Làm tương tự như vậy , ta có : y = 1 ; z = 1

Vậy .... 

5 tháng 2 2019

Sai rồi bạn , nếu làm như bạn , phải giả sử 

\(\ge y\ge x\)chứ 

:v 

12 tháng 8 2020

phải là tìm các số x,y,z thỏa mãn chứ bạn

12 tháng 8 2020

VÌ:    \(x^3+y^3+1-3xy=\left(x+y+1\right)\left(x^2+y^2+1-xy-x-y\right)\)

Do:    \(x^3+y^3+1-3xy\)   là 1 số nguyên tố

=>   \(\left(x+y+1\right)\left(x^2+y^2+1-xy-x-y\right)\)    là 1 số nguyên tố.

Do:   \(x+y+1>1\left(x,y\inℕ^∗\right)\)

=>   \(x^2+y^2-xy-x-y+1=1\)

<=> \(2x^2+2y^2-2xy-2x-2y+2=2\)

<=> \(\left(x-y\right)^2+\left(x-1\right)^2+\left(y-1\right)^2=2\)

Do:   \(\left(x-y\right)^2;\left(x-1\right)^2;\left(y-1\right)^2\)    đều là các số chính phương.

=> Ta xét 3 trường hợp sau: 

\(\hept{\begin{cases}\left(x-y\right)^2=0\\\left(x-1\right)^2=1\\\left(y-1\right)^2=1\end{cases}}\)   ;     \(\hept{\begin{cases}\left(x-y\right)^2=1\\\left(x-1\right)^2=0\\\left(y-1\right)^2=1\end{cases}}\)    ;       \(\hept{\begin{cases}\left(x-y\right)^2=1\\\left(x-1\right)^2=1\\\left(y-1\right)^2=0\end{cases}}\)

Do: x; y thuộc N* 

=> vs TH1 được: \(x=y=2\)

THỬ LẠI THÌ: \(x^3+y^3+1-3xy=8+8+1-12=5\)       (CHỌN)

TH2; TH3 tương tự ra       \(x=1;y=2\)   và     \(x=2;y=1\)

THỬ LẠI        \(\orbr{\begin{cases}x^3+y^3+1-3xy=1^3+2^3+1-3.1.2=4\\x^3+y^3+1-3xy=2^3+1^3+1-3.2.1=4\end{cases}}\)             (ĐỀU LOẠI HẾT).

VẬY \(x=y=2\)     là nghiệm duy nhất.